2014-2015/1S/DS/DC_02/DC_02.tex
2017-06-16 09:48:07 +03:00

645 lines
31 KiB
TeX

\documentclass[a4paper,10pt, table]{/media/documents/Cours/Prof/Enseignements/2014-2015/Archive/2014-2015/tools/style/classExamen}
\usepackage{/media/documents/Cours/Prof/Enseignements/2014-2015/Archive/2014-2015/2014_2015}
% Title Page
\titre{}
% \seconde \premiereS \PSTMG \TSTMG
\classe{Première S}
\date{10 février 2015}
\duree{3 heures}
%\sujet{%{{infos.subj%}}}
% DS DSCorr DM DMCorr Corr
\typedoc{Devoir Commun}
\ptpres{4}
\printanswers
\begin{document}
\titlepage
\begin{questions}
\question[9]
\begin{parts}
%1
\part Tracer un repère orthonormé et placer les points $A(-3;2)$, $B(-2;-3)$ et $C(8;1)$.
\begin{solution}
\begin{center}
\begin{tikzpicture}[scale=0.6]
\draw (-5,-5) grid (10,10);
\repere{-5}{10}{-5}{10}
\draw (-3,2) node {$\bullet$} node [above right] {$A$};
\draw (-2,-3) node {$\bullet$} node [above right] {$B$};
\draw (8,1) node {$\bullet$} node [above right] {$C$};
\end{tikzpicture}
\end{center}
\end{solution}
%2
\part Tracer la droite $(BC)$ puis calculer une équation cartésienne de cette droite.
\begin{solution}
Pour répondre à cette question, il y a deux méthodes.
\begin{itemize}
\item \textbf{Méthode 1} Où l'on cherche un vecteur directeur qui nous permet d'avoir la première partie de l'équation puis un point de cette droite permet de retrouver $c$.
\item \textbf{Méthode 2} Où l'on utilise 2 vecteurs directeurs dont un avec un point fixe puis la formule de la colinéarité permet de trouver l'équation.
\end{itemize}
Ces deux méthodes vont être faites ici. Les élèves de premières S2 sont invités à prendre connaissance de la 2e méthode, on l'utilisera plus tard dans un future chapitre.
\textbf{Méthode 1}\\
Calcul de l'équation cartésienne de la droite $\left( BC \right)$.
On commence par déterminer les coordonnées d'un vecteur directeur de $\left( BC \right)$
\begin{eqnarray*}
\vec{BC} & = & \vectCoord{x_C - x_B}{y_C - y_B} = \vectCoord{8 - (-2)}{1 - (-3)} = \vectCoord{10}{4}
\end{eqnarray*}
Donc on en déduit l'équation cartésienne de $\left( BC \right)$
\begin{eqnarray*}
4x - 10y + c & = & 0
\end{eqnarray*}
Il reste à trouver $c$ pour cela on utilise le point $C$
\begin{eqnarray*}
C(8,1) \in \left( BC \right) &\equiv & 4\times 8 - 10 \times 1 + c = 0\\
&\equiv& c = -22
\end{eqnarray*}
Donc une équation cartésienne de $\left( BC \right)$ est
\begin{eqnarray*}
4x - 10y - 22 & = & 0
\end{eqnarray*}
On peut vérifer que l'on s'est pas trompé en vérifiant que $B$ est bien un point de cette droite
\begin{eqnarray*}
4\times(-2) - 10\times(-3) - 22 & = & -8 + 30 - 22 = 0
\end{eqnarray*}
Le point $B$ est donc bien un point de cette droite.
\textbf{Méthode 2}\\
Comme pour la méthode 1, on retrouve les coordonnées du vecteur $\vec{BC} \vectCoord{10}{4}$.
On prend un point quelconque $M(x;y)$ de la droite $(BC)$ et on calcule les coordonnées de $\vec{BM}$:
\begin{eqnarray*}
\vec{BM} & = & \vectCoord{x_M - x_B}{y_M - y_B} = \vectCoord{x - (-2)}{y-(-3)} = \vectCoord{x + 2}{y+3}
\end{eqnarray*}
Comme $\vec{BC}$ et $\vec{BM}$ sont colinéaires, on sait que $xy' - x'y = 0$
\begin{eqnarray*}
4(x+2) - 10(y+3) = 0 & \equiv & 4x + 8 - 10y - 30 = 0 \\
&\equiv& 4x - 10y - 22 = 0
\end{eqnarray*}
On retrouve bien la même équation qu'avec la méthode 1. Même si ce ne sera pas fait, cette méthode s'applique aux questions suivantes.
\end{solution}
\part
\begin{subparts}
%1
\subpart Tracer la droite $(d_1)$ passant par $A$ et qui a pour vecteur directeur $\vec{v} \vectCoord{5}{2}$.
%2
\subpart Déterminer une équation cartésienne de la droite $(d_1)$.
\begin{solution}
Comme $\vec{v}$ est un vecteur directeur de $\left( d_1 \right)$, on a la première partie de son équation cartésienne
\begin{eqnarray*}
2x - 5y + c & = & 0
\end{eqnarray*}
On utilise le point $A$ pour déterminer $c$
\begin{eqnarray*}
A(-3;2)\in \left( d_1 \right) & \equiv & 2\times(-3) - 5\times 2 + c = 0 \\
&\equiv& c = 16
\end{eqnarray*}
Donc une équation cartésienne de $\left( d_1 \right)$ est
\begin{eqnarray*}
2x - 5y + 16 & = & 0
\end{eqnarray*}
\end{solution}
\end{subparts}
%1
\part Les droites $(BC)$ et $(d_1)$ sont-elles parallèles?
\begin{solution}
Ces droites sont parallèles si leurs vecteurs directeurs sont colinéaires.
\begin{eqnarray*}
xy' - x'y & = & 10\times2 - 4\times5 = 20-20 = 0
\end{eqnarray*}
Donc leurs vecteurs directeurs sont colinéaires donc les droites $\left( BC \right)$ et $\left( d_1 \right)$ sont parallèles.
\end{solution}
%2
\part Soit $(d_2)$ la droite d'équation $3x - 4y + 3 = 0$. Les droites $(d_1)$ et $(d_2)$ sont-elles parallèles?
\begin{solution}
On commence par déterminer un vecteur directeur,$\vec{w}$, de la droite $\left( d_2 \right)$:
\begin{eqnarray*}
\vec{w} = \vectCoord{-b}{a} = \vectCoord{4}{3}
\end{eqnarray*}
On vérifie si $\vec{v}$ et $\vec{w}$ sont colinéaires
\begin{eqnarray*}
xy' - x'y & = & 4\times2 - 3\times5 = 12 - 15 = -3 \neq 0
\end{eqnarray*}
Donc les vecteurs directeurs ne sont pas colinéaires donc les droites $\left( d_1 \right)$ et $\left( d_2 \right)$ ne sont pas parallèles.
\end{solution}
\end{parts}
\question[4]
Soit $f$ la fonction définie par
\begin{eqnarray*}
f:x& \mapsto & 2x^3 + 21x^2 - 180x + 6
\end{eqnarray*}
Tracer le tableau de variation de cette fonction.
\begin{solution}
Pour tracer le tableau de variation de $f$, il faut commencer par la dériver.
\begin{eqnarray*}
f'(x) & = & 2\times3\times x^2 + 21\times2 \times x - 180 \\
f'(x) & = & 6 x^2 + 42 x - 180
\end{eqnarray*}
Ensuite on étudie le signe de $f'$, pour cela on utilise la méthode du discriminant
\begin{eqnarray*}
\Delta & = & b^2 - 4ac = 42^2 - 4\times6\times(-180) = 1764 + 4320 = 6084
\end{eqnarray*}
Ici $\Delta > 0$, il y a donc deux racines
\begin{eqnarray*}
x_1 & = & \frac{-b - \sqrt{\Delta}}{2a} = \frac{-42 - \sqrt{6084}}{2\times6} = -10 \\
x_2 & = & \frac{-b + \sqrt{\Delta}}{2a} = \frac{-42 + \sqrt{6084}}{2\times6} = 3
\end{eqnarray*}
Comme $a = 6 > 0$, on en déduit le tableau de signe de $f'$ puis les variations de $f$
\begin{center}
\begin{tikzpicture}
\tkzTabInit[espcl=2]%
{$x$/1, $f'(x)$/1, $f(x)$/3}%
{$-\infty$, -10, 3, $+\infty$}
\tkzTabLine{, +, z, -, z , +,}
\tkzTabVar{-/{}, +/{$f(-10)$}, -/{$f(3)$}, +/{}}
\end{tikzpicture}
\end{center}
Avec
\begin{eqnarray*}
f(-10) & = & 2\times(-10)^3 + 21\times(-10)^2 - 180\times (-10) + 6 = 1906\\
f(3) & = & 2\times3^3 + 21\times3^2 - 180\times 3 + 6 = -291
\end{eqnarray*}
\end{solution}
\question[8]
% depuis l'exo 69 p 258
Une machine déverse du caoutchouc de façon continue dans un moule pour fabriquer des joints d'étanchéité que l'on utilise dans l'industrie automobile. On veut contrôler la régularité de l'écoulement du caoutchouc dont les variations affectent les dimensions du joint. On effectue alors des mesures sur cette machine. On obtient des masses de caoutchouc en grammes, chacune étant obtenue par un écoulement de caoutchouc d'une durée de 30~secondes. On a obtenu 20 mesures.
\begin{center}
\begin{tabular}{|*{5}{c|}}
\hline
269,7 &263,6&264,4&259,7&262,4 \\
\hline
263,4&260,7&265&267&265,6\\
\hline
268,8&260,3&263,4&267,6&264,1 \\
\hline
272,9&264,5&266,2&265,9&265,3 \\
\hline
\end{tabular}
\end{center}
\begin{parts}
\part Déterminer la moyenne, l'écart-type, la médiane, les quartiles et l'écart interquartile de cette série.
\begin{solution}
Cette question pouvait être faite avec la calculatrice.
\begin{center}
\includegraphics[scale=0.8]{./fig/calc1}
\includegraphics[scale=0.8]{./fig/calc2}
\end{center}
Voici les résultats donnés par un tableur
\begin{center}
\includegraphics[scale=0.5]{./fig/stat_corr}
\end{center}
La seule différence entre les 2 est $Q_3$. Dans la suite, on choisira la valeur de la calculatrice.
\end{solution}
%Voir le tableur pour les valeurs
\part Construire un diagramme en boîte permettant une première analyse. \label{part:diag_boite}
\begin{solution}
Diagramme en boite de cette analyse:
\begin{center}
\begin{tikzpicture}[xscale = 0.7]
\tkzInit[xmin=255,xmax=274,xstep=1]
\boxplot{1}{259.7}{263.4}{264.75}{266.6}{272.9}
\foreach \x in {255,256,...,274} \draw(\x,0)node[rotate=90] {$-$} node[below]{\x};
\draw[->] (255,0) -- (275,0);
\end{tikzpicture}
\end{center}
\end{solution}
\part Quel pourcentage des valeurs obtenues lors de ce contrôle se trouvent entre 263,4 et 266,4? %entre Q1 et Q3 vérification par le tableur.
\begin{solution}
\textbf{Avec les valeurs du tableur} on peut répondre de cette façon:\\
On remarque que $Q_1 = 263,4$ et $Q_3 = 266,4$, donc entre ces deux valeurs, il y a 50\% des données.
\textbf{Avec les valeurs de la calculatrice} on peut compter les données entre ces deux valeurs puis calculer la fréquence.
\end{solution}
\part Le statisticien J.W Tukey qualifiait d'aberrantes les valeurs d'une série statistique qui se situaient à l'extérieur de l'intervalle: $\intFF{Q_1 - 1,3\times I}{Q_3 + 1,3\times I}$, où \textbf{$I$ désigne l'écart interquartile}, $Q_1$ le premier quartile et $Q_3$ le troisième quartile.
\begin{subparts}
\subpart Le contrôle sur la machine fait-il apparaître des valeurs aberrantes? Lesquelles?
\begin{solution}
On commence par calculer les bornes de cet intervalle. Pour cela il faut calculer $I$ l'écart interquartile
\begin{eqnarray*}
I & = & Q_3 - Q_1 = 266,6 - 263,4 = 3,2
\end{eqnarray*}
Puis les bornes
\begin{itemize}
\item Borne inferieur : $Q_1 - 1,3\times I = 263,4 - 1,3\times 3,2 =259,1$
\item Borne supérieur : $Q_3 + 1,3\times I = 266,4 + 1,3\times 3,2 =270,8$
\end{itemize}
On peut compter seulement une valeur aberrante qui est $272,9$.
\end{solution}
\subpart Quel est le pourcentage de valeurs aberrantes?
\begin{solution}
Pourcentage de valeurs aberrantes:
\begin{eqnarray*}
\frac{1}{20} & = & 0,05 = 5\%
\end{eqnarray*}
\end{solution}
\end{subparts}
\part Il existe plusieurs méthodes de construction d'un diagramme en boîte. Voici une forme, légèrement différente de celle vu en cours, utilisée par J.W. Tukey pour faire apparaître les valeurs qualifiées d'aberrantes:
\begin{itemize}
\item Le rectangle central est inchangé: la limite inférieure est fixée au premier quartile, la limite supérieure au troisième quartile et une ligne, donnant la position de la médiane, coupe le rectangle.
\item Les "moustaches" sont elles modifiées: leur longueur vaut 1,3 fois l'écart interquartile. Ainsi, les deux extrémités des "moustaches" ont pour valeurs: $Q_1 - 1,3\times I$ et $Q_3 + 1,3\times I$.
\item Les valeurs situées hors de ces "moustaches" correspondent aux observations aberrantes et sont représentées chacune par un point.
\end{itemize}
\begin{subparts}
\subpart En utilisant la même graduation qu'à la question~\ref{part:diag_boite}, construire un tel diagramme en boîte correspondant aux valeurs de la série.
\begin{solution}
\begin{center}
\begin{tikzpicture}[xscale = 0.7]
\tkzInit[xmin=255,xmax=274,xstep=1]
\boxplotNoNames{1}{259.5}{263.4}{264.75}{266.4}{270.3}
\draw (272.9,1) node {$\bullet$};
\foreach \x in {255,256,...,274} \draw(\x,0)node[rotate=90] {$-$} node[below]{\x};
\draw[->] (255,0) -- (275,0);
\end{tikzpicture}
\end{center}
\end{solution}
\subpart Comparer les deux diagrammes.
\begin{solution}
On remarque que le corps du diagramme en boite n'a pas changé. Seul la moustache de droite a été modifiée.
\end{solution}
\end{subparts}
\end{parts}
%\question[8]
%Un technicien est chargé de réparer les ordinateurs. Les composants à l'origine de la panne peuvent être uniquement l'alimentation, la carte graphique ou le processeur. Une panne simultanée de deux ou trois composants est possible.
%Le technicien établie un diagnostique d'un ordinateur à l'aide d'un triplet utilisant les initiales des composants surmonté d'une barre en cas de panne. Voici deux diagnostiques possibles:
%\begin{itemize}
% \item $(A;CG;\bar{P})$ signifie que l'alimentation et la carte graphique marchent mais que le processeur est en panne.
% \item $(\bar{A};CG;\bar{P})$ signifie que la carte graphique marche mais que l'alimentation et le processeur sont en panne.
%\end{itemize}
%\begin{parts}
% \part Établir la liste des diagnostiques possibles sur un ordinateur.
% \fullwidth{ Dans la suite, on supposera que ces diagnostiques ont la même probabilité d'être établis}
% \part Le tableau suivant donne le coût des composants à remplacer
% \begin{center}
% \begin{tabular}{|c|*{3}{c|}}
% \hline
% Composant & Alimentation & Carte graphique & Processeur \\
% \hline
% Prix (en euros) & 80 & 160 & 80 \\
% \hline
% \end{tabular}
% \end{center}
% Il faut ajouter 25\euro de main-d'oeuvre (forfait indépendant du nombre de composants à remplacer) au coût des composants pour obtenir le coût de réparation.
% Soit $X$ la variable aléatoire qui, à chaque ordinateur en panne associe le coût de la réparation.
% Donner la loi de probabilité de $X$.
% \part Calculer l'espérance mathématique de $X$. Commenter le résultat.
% \part Calculer l'écart-type de $X$.
% \part Quel devrait être le coût du forfait de main-d'oeuvre, arrondi à l'unité, pour que le prix moyen d'une réparation soit de 200\euro?
%\end{parts}
\clearpage
\question[10]
L'entreprise \textit{Bibuild} veut lancer une nouvelle gamme de visseuse sur le marché. Elle a fait appel à une entreprise experte qui lui a fait les rapports suivants.
\fullwidth{
\fbox{
\parbox{\textwidth}{
\textbf{Étude de marché:}\\
Dans ce rapport, $x$ représente le prix d'une visseuse. Ce prix est limité à 130\euro\; après quoi l'étude suivante perd son sens.
Notre étude nous a amené à modéliser l'offre et la demande par les deux fonctions suivantes:
\begin{minipage}{.45\textwidth}
La fonction d'offre est donnée par
\begin{eqnarray*}
O(x) & = & -0,001x^2 + 2,3x
\end{eqnarray*}
La fonction de demande est donnée par
\begin{eqnarray*}
D(x) & = & 0,01x^2 - 2,6x + 350
\end{eqnarray*}
\end{minipage}
\hspace{0.5cm}
\begin{minipage}{0.45\textwidth}
\includegraphics[scale=0.8]{./fig/offre_demande}
\end{minipage}
\\
Nous vous rappelons que pour qu'une entente soit trouvée entre l'acheteur et le vendeur, il faut que l'offre soit égale à la demande. On appelle alors \textit{prix d'équilibre du produit} le prix pour lequel l'offre est égale à la demande.
}}}
\fullwidth{
\fbox{
\parbox{\textwidth}{
\textbf{Étude de la production}\\
Ici $x$ représente la quantité de visseuses. Les capacités de l'usine font que la production est limitée à 300 visseuses.
\begin{minipage}{0.4\textwidth}
Coût de production
\begin{eqnarray*}
C(x) & = & 0,3x^2 + 3x + 3000
\end{eqnarray*}
\end{minipage}
\begin{minipage}{0.45\textwidth}
\includegraphics[scale=0.8]{./fig/cout}
\end{minipage}
\\
Nous vous rappelons que les bénéfices se calculent de la manière suivante
\begin{center}
Bénéfices = Recettes - Coûts
\end{center}
}}}
\clearpage
\fullwidth{
\textbf{Analyse de l'étude de marché}
L'étude de marché va permettre de déterminer le prix optimal d'une visseuse.
}
\begin{parts}
\part
\begin{subparts}
\subpart Déterminer \textbf{graphiquement} le nombre de produits offerts et le nombre de produits demandés lorsque le prix du produit est de 18\euro.
\begin{solution}
À cette question, il est mal vu de reprendre les résultats du calcul de la question suivante surtout quand la précision du graphique ne le permet pas ou que les calculs sont faux...
\end{solution}
\subpart Retrouver ces résultats par le calcul.
\begin{solution}
Nombre de produits offerts
\begin{eqnarray*}
O(18) & = & -0,001\times18^2 + 2,3\times 18 \approx 41
\end{eqnarray*}
Nombre de produits demandés
\begin{eqnarray*}
D(18) & = & 0,01\times18^2 - 2,6\times 18 + 350 \approx 306
\end{eqnarray*}
\end{solution}
\subpart Dans ce cas là, y a-t-il plus d'offre ou de demande?
\begin{solution}
Pour des visseuses à 18\euro la demande est plus forte que l'offre.
\end{solution}
\subpart Donner un prix où la situation est inversée.
\begin{solution}
Si une visseuse coûte 110\euro, l'offre serait plus forte que la demande.
\end{solution}
\end{subparts}
\part Pour que l'entreprise puisse vendre tout ce qu'elle produit, il faut qu'elle fixe le prix de ces visseuses au prix d'équilibre. Déterminer ce prix et la quantité échangée associée.
\begin{solution}
On cherche le prix, $x$, pour que l'offre, $O(x)$, soit égale à la demande, $D(x)$. On résout donc l'équation suivante:
\begin{eqnarray*}
O(x) = D(x) & \equiv & -0,001x^2 + 2,3x = 0,01x^2 - 2,6x + 350 \\
& \equiv & -0,001x^2 - 0,01x^2 + 2,3x + 2,6x - 350 = 0 \\
& \equiv & -0,011x^2 + 4,9x - 350 = 0
\end{eqnarray*}
On reconnait une équation de 2nd degré.
\begin{eqnarray*}
\Delta & = & b^2 - 4ac\\
&=& 4,9^2 - 4 \times (-0,011) \times (-350) \\
&=& 8,61
\end{eqnarray*}
$\Delta$ est positif, il y a donc deux solutions
\begin{eqnarray*}
x_1 & = & \frac{-b-\sqrt{\Delta}}{2a} = \frac{-4,9 - \sqrt{8,61}}{2\times(-0,011)} = 356.10\\
x_2 & = & \frac{-b+\sqrt{\Delta}}{2a} = \frac{-4,9 + \sqrt{8,61}}{2\times(-0,011)} = 89.35\\
\end{eqnarray*}
La première solution ne correspond pas à un prix acceptable pour l'expert car $x$ est limité entre 0 et 130.
Donc le prix d'équilibre est donc 89,35\euro.
\end{solution}
\fullwidth{
\textbf{Analyse de la production}
Le gérant de l'entreprise, après avoir lu l'étude de marché, fixe le prix d'une visseuse à 89,99\euro. Il doit maintenant déterminer combien de visseuses son entreprise doit produire pour maximiser ses bénéfices.
}
\part Déterminer la fonction $R$ qui calcule les recettes à partir du nombre de visseuses $x$.
\begin{solution}
Comme chaque visseuse est vendue 89,99\euro, les recettes sont données par la fonction suivante
\begin{eqnarray*}
R(x) & = & 89,99x
\end{eqnarray*}
\end{solution}
\part Combien doit-il produire de visseuses pour que ses bénéfices soient maximaux?
\begin{solution}
On commence par déterminer la fonction bénéfice
\begin{eqnarray*}
B(x) &= & R(x) - C(x) \\
&=& 89,99x - (0,3x^2 + 3x + 3000) \\
&=& -0,3x^2 + 86,99x - 3000
\end{eqnarray*}
Pour déterminer le nombre de visseuses pour que les bénéfices soient maximaux il faut avoir le tableau de variation de $B$. Pour cela il y a deux méthodes.
\begin{itemize}
\item En dérivant $B$ puis en étudiant le signe de $B'$ pour trouver le sens de variation de $B$.
\item En utilisant le chapitre sur la forme canonique. C'est cette méthode qui va être détaillé ici.
\end{itemize}
On calcule les coordonnées du sommet de la parabole
\begin{eqnarray*}
\alpha & = & \frac{-b}{2a} = \frac{-86,99}{2\times(-0,3)} \approx 144,98\\
\beta & = & -\frac{b^2 - 4ac}{4a} = 3306,05
\end{eqnarray*}
Comme $a = -0,3 < 0$ on en déduit le tableau de variation
\begin{center}
\begin{tikzpicture}
\tkzTabInit[espcl=4,lgt=3]%
{$x$/1, {Variation de \\ $B(x)$}/3}%
{$-\infty$, {$\alpha = 144,98$}, $+\infty$}
\tkzTabVar{-/{}, +/{$\beta = 3306,05$}, -/{}}
\end{tikzpicture}
\end{center}
Comme un nombre de visseuse est un nombre entier, il faut choisir entre $x=144$ et $x = 145$.
\begin{eqnarray*}
B(144) & = & -0,3\times144^2 + 86,99\times 144 - 3000 = 3305,76\\
B(145) & = & -0,3\times145^2 + 86,99\times 145 - 3000 = 3306,04
\end{eqnarray*}
Les bénéfices sont maximaux pour 145 visseuses vendues. L'entreprise fait alors 3306,04\euro de bénéfices.
\end{solution}
\part Combien de zavisseuses doit-il produire pour que ses bénéfices soient supérieurs à 3000\euro?
\begin{solution}
Dans cette question, toute trace de recherche était valorisée. Voici quelques possibilités pour approcher le résultat.
\begin{itemize}
\item Faire un tableau de valeur avec la calculatrice et déterminer les quantités
\item Faire un graphique et retrouver ces nombre.
\item Résoudre l'inéquation $B(x) > 3000$. C'est cette méthode qui est utilisée ici.
\end{itemize}
Pour que les bénéfices soient supérieurs à 3000\euro, il faut trouver les valeurs de $x$ tels que $B(x) > 3000$ ou encore résoudre l'équation suivante
\begin{eqnarray*}
-0,3x^2 + 86,99x - 3000 > 3000 &\equiv & -0,3x^2 + 86,99x - 6000> 0
\end{eqnarray*}
On utilise la méthode du discriminant.
\begin{eqnarray*}
\Delta & = & b^2 - 4ac = 86,99^2 - 4\times (-0,3) \times (-6000) \\
& = & 367.26
\end{eqnarray*}
Ici $\Delta > 0$, il y a donc deux racines
\begin{eqnarray*}
x_1 & = & \frac{-b - \sqrt{\Delta}}{2a} = \frac{-86,99 - \sqrt{367,26}}{2\times(-0,3)} = 176,92 \\
x_2 & = & \frac{-b + \sqrt{\Delta}}{2a} = \frac{-86,99 + \sqrt{367,26}}{2\times(-0,3)} = 113,04 \\
\end{eqnarray*}
Ici $a = -0,3 < 0$ on en déduit le tableau de signe suivant
\begin{tikzpicture}
\tkzTabInit[espcl=3, lgt=4]%
{$x$/1, {Signe de \\$-0,3x^3 + 86,99x - 6000$}/2}%
{$-\infty$, 0, 2, $+\infty$}
\tkzTabLine{, -, z, +, z, -,}
\end{tikzpicture}
Donc les bénéfices sont supérieurs à 3000\euro quand la production est comprise entre 114 et 176 visseuses.
\end{solution}
\end{parts}
\clearpage
\question[5]
\begin{itshape}
Cet exercice est un questionnaire à choix multiplies (QCM).
Pour chaque question, une seule des trois réponses proposées est correcte.
Indiquer sur la copie le numéro de la question ainsi que la réponse choisie. Aucune justification n'est demandée.
Une réponse juste rapporte 1~point, une réponse fausse ou l'absence de réponse ne rapporte ni n'enlève de point. Si le total des points est négatif, la note attribuée à l'exercice est ramenée à 0.
\end{itshape}
\hspace{-1cm}
\begin{tabular}{|c|m{5.5cm}|*{3}{>{\centering\arraybackslash}m{3.5cm}|}}
\hline
&& A & B & C \\
\hline
1&Placer l'angle $\alpha = \dfrac{-5\pi}{4}$. &
\ifprintanswers
\cellcolor{green}
\fi
\begin{tikzpicture}[baseline={([yshift=-.8ex]current bounding box.center)}]
\cercleTrigo
\draw (135:1) node[rotate=135] {$-$} node[above left] {$\alpha$};
\end{tikzpicture}
&
\begin{tikzpicture}[baseline={([yshift=-.8ex]current bounding box.center)}]
\cercleTrigo
\draw (225:1) node[rotate=225] {$-$} node[below left] {$\alpha$};
\end{tikzpicture}
&
\begin{tikzpicture}[baseline={([yshift=-.8ex]current bounding box.center)}]
\cercleTrigo
\draw (90:1) node[rotate=90] {$-$} node[above right] {$\alpha$};
\end{tikzpicture}
\\
\hline
2&Donner la mesure principale de l'angle $\dfrac{2015\pi}{4}$ &
\ifprintanswers
\cellcolor{green}
\fi
$\dfrac{-1}{4}\pi$ & %Good one
$\dfrac{7}{4}\pi$ &
$503,75\pi$ \\
\hline
3&Convertir en degrés $\frac{-5\pi}{3}$ &
\ifprintanswers
\cellcolor{green}
\fi
300& % Good one
-300&
60\\
\hline
4&Calculer le nombre dérivé en -3 de $f:x\mapsto x^2 - 4x - 1$ &
\ifprintanswers
\cellcolor{green}
\fi
-10 & %Good one
$10 + h$&
-2 \\
\hline
5&Déterminer l'équation réduite de la tangente en 1 de la fonction suivante
\begin{tikzpicture}[scale=0.8]
\repere{-2}{4}{-1}{4}
\draw[very thick, domain=-2:4, color=red] plot [samples=300] (\x, {cos(pi*deg(\x)/2) + 1});
\clip (-2,-1) rectangle (4,4);
\end{tikzpicture}
&
\ifprintanswers
\cellcolor{green}
\fi
$y = -x + 2$&
$y = x + 2$&
$y = 2x + 1$\\
\hline
\end{tabular}
\begin{solution}
On ne demandait de justifier le QCM mais voici les explications.
\begin{enumerate}
\item La réponse $B$ correspond à un angle de $\frac{5\pi}{4}$ et la réponse $C$ correspond à un angle de $\frac{\pi}{2}$.
\item
\begin{eqnarray*}
\frac{2015\pi}{4} = \frac{-\pi}{4} + \frac{2016\pi}{4} = \frac{-\pi}{4} + 252\pi
\end{eqnarray*}
$252\pi$ correspond à 126 tours complets. La mesure principale de l'angle $\frac{2015\pi}{4}$ est donc $\frac{-\pi}{4}$ qui est bien dans $\intOF{-\pi}{\pi}$.
\item Pour convertir en degré, on peut faire un produit en croix ou directement le calcul suivant
\begin{eqnarray*}
\frac{-5\pi}{3} \times \frac{180}{\pi} & = & \frac{-5\times 180}{3} = -300
\end{eqnarray*}
Or un angle en degré n'est pas orienté donc il n'a pas de signe. La réponse est donc $300$.
\item Pour calculer le nombre dérivé, soit on utilise le tauw d'accoissement ($\lim_{h\rightarrow 0} \frac{f(x+h) - f(x)}{h}$ soit on calcule la dérivé. C'est cette dernière méthode qui va être présentée.
\begin{eqnarray*}
f'(x) & = & 2x - 4 + 0
\end{eqnarray*}
On évalue en -3 la dérivé pour trouver le nombre dérivé
\begin{eqnarray*}
f'(-3) & = & 2\times(-3) - 4 = -10
\end{eqnarray*}
\item On trace la tangente à la courbe en 1
\begin{tikzpicture}[scale=0.8]
\repere{-2}{4}{-1}{4}
\clip (-2,-1) rectangle (4,4);
\draw[very thick, domain=-2:4, color=red] plot [samples=300] (\x, {cos(pi*deg(\x)/2) + 1});
\draw (-2,4) -- (3,-1);
\end{tikzpicture}
\end{enumerate}
\end{solution}
\end{questions}
\end{document}
%%% Local Variables:
%%% mode: latex
%%% TeX-master: "master"
%%% End: